Improper integral question

ozgunozgur

New member
Joined
Apr 1, 2020
Messages
37
sdsdsdsdsds.png

This is my method, could you help me to continue?

< link to imgur's 'Most Viral Images' removed >
 
Your integrand is proportional to [math]1/\sqrt{x}[/math] (think "asymptotic behavior"), whose integral is proportional to [math]\sqrt{x}[/math]. This does not converge as the input is taken to infinity.
 
daon gave a nice hint.

Now why are you not finishing? The integrals you need to solve for are easy ones. What is troubling you? You do know that even if one of those integrals diverges then you are done? Continue...
 
Top